Sie sind auf Seite 1von 46

Homework Solutions for Engineering Economic Analysis, 10th Edition

Newnan, Lavelle, Eschenbach

Chapter 7: Rate of Return Analysis


7-1
$100 = $27 (P/A, i%, 10)
(P/A, i%, 10) = 3.704
Performing Linear Interpolation:
(P/A, i%, 10) I
4.192
20%
3.571
25%
Rate of Return = 20% + (5%) [(4.192 3.704)/(4.912 3.571)]
= 23.9%

7-2
(a) F = $5, P = $1, n = 5
F = P (1 + i)n
$5 = $1 (1 + i)5
(1 + i) = 50.20 = 1.38
i* = 38%
(b) For a 100% annual rate of return
F = $1 (1 + 1.0)5 = $32, not $5!
Note that the prices Diagonal charges do not necessarily reflect what anyone will
pay a collector for his/her stamps.

Homework Solutions for Engineering Economic Analysis, 10th Edition


Newnan, Lavelle, Eschenbach

7-3
A = $12.64
.
n = 12
$175

($175 $35) = $12.64 (P/A, i%, 12)


(P/A, i%, 12) = $140/$12.64 = 11.08
i = 1 %
Nominal interest rate = 12 (1 %) = 15%

7-4
The rate of return exceeds 60% so the interest tables are not useful.
F = P (1 + i)n
$25,000 = $5,000 (1 + i)3
(1 + i) = ($25,000/$5,000)1/3 = 1.71
i* = 0.71
Rate of Return = 71%

7-5
$3,000
A = $325

n = 36

$12,375

$9,375 = $325 (P/A, i%, 36)


(P/A, i%, 36) = $9,375/$325 = 28.846
From compound interest tables, i = 1.25%
Nominal Interest Rate = 1.25 12 = 15%
Effective Interest Rate = (1 + 0.0125)12 1 = 16.08%
2

Homework Solutions for Engineering Economic Analysis, 10th Edition


Newnan, Lavelle, Eschenbach

7-6
1991 1626 = 365 years = n
F = P (1 + i)n
12 109 = 24(1 + i)365
(1 + i)365 = 12 x 100/24= 5.00 108
This may be immediately solved on most hand calculators:
i* = 5.64%
Solution based on compound interest tables:
(F/P, i%, 365) = 5.00 108
= (F/P, i%, 100) (F/P, i%, 100) (F/P, i%, 100) (F/P, i%, 65)
Try i = 6%
(F/P, 6%, 365) = (339.3)3 (44.14) = 17.24 108 (i too high)
Try i = 5%
(F/P, 5%, 365) = (131.5)3 (23.84) = 0.542 108 (i too low)
Performing linear interpolation:
i* = 5% + (1%) [((5 0.54) (108))/((17.24 0.54) (108))]
= 5% + 4.46/16.70
= 5.27%
The linear interpolation is inaccurate.

7-7
106
(F/A, i, 35) =
= 172.414 and is very close to 8% from tables. (Exact = 8.003%)
5800

7-8
107
= 40 and interpolating
2.5 105
36.786 40
i = 6% + (1%)
= 6.76% (exact value 6.774%)
36.786 40.996
(F/A, i, 20) =

Homework Solutions for Engineering Economic Analysis, 10th Edition


Newnan, Lavelle, Eschenbach

7-9
Year
0
3
6

Cash Flow
$1,000
+$1,094.60
+$1,094.60

$1,000 = $1,094 [(P/F, i%, 6) + (P/F, i%, 9)]


Try i = 20%
$1,094 [(0.5787) + (0.3349)] = $1,000
Rate of Return = 20%

7-10
3,000 = 30 (P/A, i*, 120)
(P/A, i*, 120) = 3,000/30 = 100
Performing Linear Interpolation:
(P/A, i%, 120)
103.563
100
90.074

I
%
i*
%

i* = 0.0025 + 0.0025 [(103.562 100)/(103.562 90.074)]


= 0.00316 per month
Nominal Annual Rate = 12 (0.00316) = 0.03792 = 3.79%

Homework Solutions for Engineering Economic Analysis, 10th Edition


Newnan, Lavelle, Eschenbach

7-11
$3,000 = $119.67 (P/A, i%, 30)
(P/A, i%, 30) = $3,000/$119.67 = 25.069
Performing Linear Interpolation:
(P/A, i% 30) i
25.808
1%
24.889
1.25%
i = 1% + (0.25%)((25.80825.069)/(25.80824.889))
= 1.201%
(a) Nominal Interest Rate = 1.201 12 = 14.41%
(b) Effective Interest Rate = (1 + 0.01201)12 1 = 0.154 = 15.4%

7-12
$125

$10

$20

$30

$40

$50

$60

$125 = $10 (P/A, i%, 6) + $10 (P/G, i%, 6)


at 12%, $10 (4.111) + $10 (8.930) = $130.4
at 15%, $10 (3.784) + $10 (7.937) = $117.2
i* = 12% + (3%) ((130.4 125)/(130.4117.2)) = 13.23%

Homework Solutions for Engineering Economic Analysis, 10th Edition


Newnan, Lavelle, Eschenbach

7-13

$5

$10

$15

$20

$25

$42.55

$42.55 = $5 (P/A, i%, 5) + $5 (P/G, i%, 5)


Try i = 15%, $5 (3.352) + $5 (5.775) = $45.64 > $42.55
Try i = 20%, $5 (2.991) + $5 (4.906) = $39.49 < $42.55
Rate of Return = 15% + (5%) [($45.64 $42.55)/($45.64 $39.49)]
= 17.51%
Exact Answer: 17.38%

7-14
The algebraic sum of the cash flows equals zero. Therefore, the rate of return is 0%.

7-15
A = $300

$1,000

Try i = 5%
$1,000 = (?) $300 (3.546) (0.9524)
= (?) $1,013.16
Try i = 6%
$1,000 = (?) $300 (3.465) (0.9434)
= (?) $980.66
Performing Linear Interpolation:
i* = 5% + (1%) (($1,013.6 $1,000)/($1,013.6 $980.66))
= 5.4%

Homework Solutions for Engineering Economic Analysis, 10th Edition


Newnan, Lavelle, Eschenbach

7-16
Since the rate of return exceeds 60%, the tables are useless.
F = P (1 + i)n
$4,500 = $500 (1 + i)4
(1 + i)4 = $4,500/$500 = 0
(1 + i) = 9 = 1.732
i* = 0.732 = 73.2%

7-17
(a) Using Equation (4-39):
F = Pem
$4,000 = $2,000er(9)
2 = er(9)
9r = In 2 = 0.693
r = 7.70%
(b) Equation (4-34)
ieff = er 1 = e0.077 1 = 0.0800 = 8.00%

7-18
Year
0
1
2
3
4
5

Cash Flow
$640
40
+$100
+$200
+$300
+$300

$640 = $100 (P/G, i%, 4) + $300 (P/F, i%, 5)


Try i = 9%
$100 (4.511) + $300 (0.6499) = $646.07 > $640
Try i = 10%
$100 (4.378) + $300 (0.6209) = $624.07 < $640
Rate of Return = 9% + (1%) [(%646.07 $640)/($646.07 $624.07)]
= 9.28%

Homework Solutions for Engineering Economic Analysis, 10th Edition


Newnan, Lavelle, Eschenbach

7-19
Year
0
1
2
3
4
5
6
7
8
9
10

Cash Flow
$223
$223
$223
$223
$223
$223
+$1,000
+$1,000
+$1,000
+$1,000
+$1,000

The rate of return may be computed by any conventional means. On closer


inspection one observes that each $223 increases to $1,000 in five years.
$223 = $1,000 (P/F, i%, 5)
(P/F, i%, 5) = $223/$1,000 = 0.2230
From interest tables, Rate of Return = 35%

7-20
Do nothing has a cash flow of zero, thus, the difference between alternatives is just
the Leaseco cash flow.
Year
0
1
2
3
4
5

Leaseco Do
Nothing
$1,000
$200
$200
$1,200
$1,200
$1,200

NPW = 0 = 1000 + 200 (P/A, ROR, 5) + 1000 (P/F, i, 2) (P/A, i, 3) and interpolating
85.271

ROR = 45% + (5%)


= 48.8%. Obviously, do nothing is rejected.
85.271 27.329

Homework Solutions for Engineering Economic Analysis, 10th Edition


Newnan, Lavelle, Eschenbach

7-21
$80 $80 $80 $80 $80 $80

$200

$200

$200

The easiest solution is to solve one cycle of the repeating diagram:


$80 $80

$80

$200

$120

$120 = $80 (F/P, i%, 1)


$120 = $80 (1 + i)
(1 + i) = $120/$80 = 1.50
i* = 0.50 = 50%
Alternative Solution:
EUAB = EUAC
$80 = [$200 (P/F, i%, 2) + $200 (P/F, i%, 4) + $200 (P/F, i%, 6)] (A/P, i%, 6)
Try i = 50%
$80 = [$200 (0.4444) + $200 (0.1975) + $200 (0.0878)] (0.5481) = $79.99
Therefore i* = 50%

Homework Solutions for Engineering Economic Analysis, 10th Edition


Newnan, Lavelle, Eschenbach

7-22
For infinite series: A = Pi
EUAC = EUAB
$3,810 (i) = $250 + $250 (F/P, i%, 1) (A/F, i%, 2)*
Try i = 10%
$250 + $250 (1.10) (0.4762) = $381
$3,810 (0.10) = $381
i = 10%
*

Alternate Equations:
$3,810 (i) = $250 + $250 (P/F, i%, 1) (A/P, i%, 2)
$3,810 (i) = $500 $250 (A/G, i%, 2)

7-23
P

A = $1,000

Yr 0
n=

n = 10
$412

$5,000

At Year 0, PW of Cost = PW of Benefits


$412 + $5,000 (P/F, i%, 10) = ($1000/i) (P/F, i%, 10)
Try i = 15%
$412 + $5,000 (0.2472) = ($1,000/0.15) (0.2472)
$1,648 = $1,648
ROR = 15%

10

Homework Solutions for Engineering Economic Analysis, 10th Edition


Newnan, Lavelle, Eschenbach

7-24
$400 = [$200 (P/A, i%, 4) $50 (P/G, i%, 4)] (P/F, i%, 1)
Try i = 7%
[$200 (3.387) $50 (4.795)] (0.9346) = 409.03
Try i = 8%
[$200 (3.312) $50 (4.650)] (0.9259) = $398.08
i* = 7% + (1%) [($409.03 $400)/($409.03 $398.04)]
= 7.82%

7-25
The one-time $2,000 life membership fee avoids the 40-year series of beginning-ofyear membership dues that start at $200 and increase 3% annually.
(a) The equation for determining the rate of return for the life membership is the
difference of the present worth of the two cash flows set to zero:
2000 200 206 (P/A, 3%, ROR, 39) = 0 (39 since beginning-of-year
payments)
1800 1 (1 0.03)39 (1 i) 39

(b) Use Excel where Result =


and vary i = ROR until
206
i 0.03

zero is obtained. ROR = 14.243%

11

Homew
work Solutions for Engin
neering Econ
nomic Analys
sis, 10th Editiion
Newnan, La
avelle, Esche
enbach

7-26
Ye
ear
0
1
2
3
4
5
6
7
8
9
10

Cash
Flow
100
27
27
27
27
27
27
27
27
27
27

PW

0%
5%
10%
15%
20%
25%
30%
35%
40%
45%
50%

170.0
108.5
65.9
35.5
13.2
-3.6
-16.5
-26.7
-34.8
-41.5
-46.9

PW = -100 + 27
7*(P/A, i, 10
0) ; use NPV
V in for (P/A
A, i, 10) in E
Excel.

estment.
This is a typical PW graph for an inve

12

Homew
work Solutions for Engin
neering Econ
nomic Analys
sis, 10th Editiion
Newnan, La
avelle, Esche
enbach

7-27
Period
Value

0
-640

1
0

Interest, i
0%
5%
10%
15%
20%
25%
30%
35%
40%
45%
50%

2
100

3
4
5
20
00 300 300
3

PW
260
105
-16
-112
-190
-252
-304
-347
-382
-412
-438

This
T
is a typical NPW graph.
g
As th
he interest rrate increasses, future benefits arre
discounted more
m
heavily and the NPW
N
decre
eases.

13

Homework Solutions for Engineering Economic Analysis, 10th Edition


Newnan, Lavelle, Eschenbach

7-28
$1,000
A = $40

n = 10
$925

PW of Cost = PW of Benefits
$925 = $40 (P/A, i%, 10) + $1,000 (P/F, i%, 10)
Try i = 5%
$925 = $40 (7.722) + $1,000 (0.6139) = $922.78 (i too high)
Try i = 4.5%
$925 = $40 (7.913) + $1,000 (0.6439) = $960.42 (i too low)
i* = 4.97%

7-29
$1,000
A = $40

n = 40 semiannual periods
$715

PW of Benefits PW of Costs = 0
$20 (P/A, i%, 40) + $1,000 (P/F, i%, 40) $715 = 0
Try i = 3%
$20 (23.115) + $1,000 (0.3066) $715 = $53.90 i too low
Try i = 3.5%
$20 (21.355) + $1,000 (0.2526) $715 = $35.30 i too high
Performing linear interpolation:
i* = 3% + (0.5%) [53.90/(53.90 (35.30))] = 3.30%
Nominal i* = 6.60%
14

Homework Solutions for Engineering Economic Analysis, 10th Edition


Newnan, Lavelle, Eschenbach

7-30
$1,000
A = $30
.
n = 2(2001 1998) + 1 = 27
$875

PW of Benefits PW of Cost = $0
$30 (P/A, i%, 27) + $1,000 (P/F, i%, 27) $875 = $0
Try i = 3 %
$30 (17.285) + $1,000 (0.3950) $875 = $38.55 >$0
Try i = 4%
$30 (16.330) + $1,000 (0.3468) $875 = $38.30 < $0
i* = 3.75%
Nominal rate of return = 2 (3.75%) = 7.5%

7-31
6.8%
= 3.4%, so (0.034) (1000) = $34 is
2
paid semi-annually and $1,000 is paid at the end of the 10th year (20th pay
period).

(a) For the cash flow of the bond have i =

NPW = 0 = +1000 34 (P/A, i, 20) 1000 (P/F, i, 20) and interpolating


59.518

= 3.404% (exact value = 3.400%),


i = 3% (0.5%)
59.518 14.192
r = (2) (3.404%) = 6.808%, and ia = (1+0.03404)2 1 = 0.06924 or 6.924%.
(b) The fee is $1,000 x 0.0075 = $7.50. So ABC Corp. receives $1,000 $7.50 =
$992.50.
NPW = 0 = 992.5 - 34 (P/A, i, 20) 1000 (P/F, i, 20) and interpolating
67.018

i = 3% (0.5%)
= 3.4546% (exact value = 3.453%),
67.018 6.692
r = (2) (3.4546%) = 6.909%, and ia = (1 + 0.034546)2 1 = 0.07029 or 7.029%.

15

Homework Solutions for Engineering Economic Analysis, 10th Edition


Newnan, Lavelle, Eschenbach

7-32
(a) NPW = 0 = -3118 + 10000 (P/F, i, 20), so, (P/F, i, 20) = 0.3118. Next you can
solve (1 i ) 20 0.3118 for i or look in the tables to find i = 0.06 or 6.0%.
Next, because it is paid annually, the effective annual interest rate is 6.0%.
(b) The fee is $10,000 x 0.01 = $100. So ABC Corp. receives $3,118 - $100 =
$3,018.
NPW = 0 = 3018 10000 (P/F, i, 20), so, (P/F, i, 20) = 0.3018. Next
(1 i) 20 0.3018 and find i = 0.06173 or 6.173%. As above ia = 6.173%.

7-33
A = $110
.
n = 24
$3,500 - $1,200
= $2,300

$2,300 = $110 (P/A, i%, 24)


(P/A, i%, 24) = $2,300/$110 = 20.91
From tables: 1 % < i < 1.25%
On Financial Calculator: i = 1.13% per month
Effective interest rate = (1 + 0.0113)12 1 = 0.144 = 14.4%

16

solve

Homework Solutions for Engineering Economic Analysis, 10th Edition


Newnan, Lavelle, Eschenbach

7-34
A = $100
.
n = 36
$3,168

PW of Cost = PW of Benefits
$100 (P/A, i%, 36) = $3,168
(P/A, i%, 36) = $3,168/$100 = 31.68
Performing Linear Interpolation:
(P/A, 1%, 36) i
32.871
%
21.447
%
i* = (1/2%) + (1/4%) [(32.87 31.68)/(32.87 31.45)]
= 0.71%
Nominal Interest Rate = 12 (0.71%) = 8.5%

7-35
F = $2,242
A = $50

n=4
P = $1,845

Set PW of Cost = PW of Benefits


$1,845 = $50 (P/A, i%, 4) + $2,242 (P/F, i%, 4)
Try i = 7%
450 (3.387) + $2,242 (0.7629) = $1,879 > $1,845
Try i = 8%

17

Homework Solutions for Engineering Economic Analysis, 10th Edition


Newnan, Lavelle, Eschenbach

450 (3.312) + $2,242 (0.7350) = $1,813 < $1,845


Rate of Return = 7% + (1%) [($1,879 $1,845)/($1,879 $1,813)]
= 7.52% for 6 months
Nominal annual rate of return = 2 (7.52%) = 15.0%
Equivalent annual rate of return = (1 + 0.0752)2 1 = 15.6%

7-36
6000
$166.67 (over 3 years).
36
NPW = 0 = 6000 -250 166.67 (P/A, i, 36), so, (P/A, i, 36) = 34.50 . The tables
dont go to a low enough interest rate so must solve:
(1 i )36 1
i (1 i )36 34.50 by trial and error or Excel using the IRR function. Excel

yields

(a) The monthly payments are

i = 0.00232, so, ia = (1 + 0.00232)12 1 = 0.0282 or 2.82%.


(b) The fact that the dealer would accept $5,200 cash for the car indicates its true
worth so the extra $800 is a hidden finance charge. Your payments are still
based on the original $6,000 cost but you only receive a car worth only $5,200!
NPW = 0 = 5200 -250 166.67 (P/A, i, 36), so, (P/A, i, 36) = 29.70 and
interpolating
30.107 29.70
i = 1% + (0.25%)
= 1.081% (exact value = 1.079%), so,
30.107 28.847
ia = (1 + 0.01081)12 1 = 13.77% (exact value = 13.75%).

18

Homework Solutions for Engineering Economic Analysis, 10th Edition


Newnan, Lavelle, Eschenbach

7-37
(a) The foregone cash rebate is like a hidden finance charge. You pay $12,000 for
the car but receive a car only worth $12,000 - $3,000 = $9,000. The monthly
12000
payments =
= $250 for 48 months. NPW = 0 = 9000 250 (P/A, i, 48),
4 12
so, (P/A, i, 48) = 36.0 and interpolating
37.974 36.0
i = 1% + (0.25%)
= 1.242%, so, r = (12) (1.242%) = 14.90%
37.974 35.932
and ia = (1 + 0.01242)12 -1 = 0.15965 or 15.97%.

(b) Worth of car = Cost Rebate = $18,000 - $3,000 = $15,000.


18000
The monthly payments =
= $375 for 48 months.
4 12
NPW = 0 = 15000 375 (P/A, i, 48), so, (P/A, i, 48) = 40.0 and interpolating
40.185 40.0
i = 0.75% + (0.25%)
= 0.771%, so, r = (12)(0.771%) = 9.65%
40.185 37.974
and ia = (1 + 0.00771)12 -1 = 0.0965 or 9.65%.
(c) Worth of car = Cost Rebate = $24,000 - $3,000 = $21,000.
24000
The monthly payments =
= $500 for 48 months.
4 12
NPW = 0 = 21000 500 (P/A, i, 48), so, (P/A, i, 48) = 42.0 and interpolating
42.580 42.0
i = 0.50% + (0.25%)
= 0.561%, so, r = (12)(0.561%) = 6.73%
42.580 40.185
and ia = (1 + 0.00561)12 -1 = 0.0694 or 6.94%.

7-38
First determine the monthly payments for the loan where i =

4%
= 0.3333%, so,
12

(0.003333)(1 0.003333)36
A = 6000 (A/P, 0.3333%, 36) =
= $177.14 .
(1 0.003333)36 1

(a) NPW = 0 = 6000 250 177.14 (P/A, i, 36), so, (P/A, i, 36) = 32.46 and
interpolating
32.871 32.46
i = 0.50% + (0.25%)
= 0.572%, so,
32.871 31.447
r = (12) (0.572%) = 6.86% and ia = (1 + 0.00572)12 1 = 0.0709 or 7.09%.

19

Homework Solutions for Engineering Economic Analysis, 10th Edition


Newnan, Lavelle, Eschenbach

(b) Worth of the car = $6,000 $800 = $5,200 but the payments are determined by
the actual cost to buyer, here $6,000. Thus, the payments are the same as
above.
NPW = 0 = 5200 250 177.14 (P/A, i, 36), so, (P/A, i, 36) = 27.944 and
28.847 27.944
= 1.440%, so,
interpolating i = 1.25% + (0.25%)
28.847 27.661
r = (12) (1.440%) = 17.28% and ia = (1 + 0.01440)12 1 = 0.1872 or 18.72%.
(c) The actual value of the car seems to be the most important factor!

7-39
The amount of cash paid will be $75,000 $50,000 = $25,000 with $50,000
financed, so, the monthly payments will be 50000 (A/P, 8%, 4) = (50000) (0.3019) =
$15,095. The reduction in cost if one pays entirely in cash is $75,000 x 0.10 =
$7,500, so, a 100% cash payment would be $75,000 $7,500 = $67,500 (true value
of equipment).
Year
0
1
2
3
4

Borrow from
Pay Cash Manufacturer
-$67,500 $25,000
15,095
15,095
15,095
15,095

Incremental
Difference
$42,500
15,095
15,095
15,095
15,095

IRR = IRR (the (1) (2) values for the Periods 04) = 15.69% per year

7-40
The loan value is $120,000 $12,000 (10% down payment) = $108,000. The loan
origination fee is $108,000 x 0.02 = $2,160, so, the loan becomes $108,000 +
$2,160 = $110,160.
6%
(a) Number of months is 30 x 12 = 360. The monthly interest rate, i =
= 0.5%.
12
The monthly payment = 110160 (A/P, 0.5%, 360) but to get accuracy use

110160
110160

$660.46 .
( P / A, 0.5%,360) 166.792

20

Homework Solutions for Engineering Economic Analysis, 10th Edition


Newnan, Lavelle, Eschenbach

(b) The actual value received is $108,000, thus, to find the effective interest rate
solve
NPW = 0 = 108,000 660.46 (P/A, i, 360).
108000
= 163.522. Interpolating
(P/A, i, 360) =
660.46
imo = % + ( %)[(163.522 166.792)/(124.282 166.792]
= 0.51923% per month
ia = (1 + 0.0051923)12 1 = 0.0641 or 6.41%
(c) In ten years there are still 20 years left on the original loan, so,
value of remaining loan at year ten
= 660.46 (P/A, 0.5%, 240) = (660.46)(139.581)
= $92,187.67 . To find the effective interest rate solve
NPW = 108,000 660.46 (P/A, i, 120) 92,187.67 (P/F, i, 120) . Interpolating
imo = % + ( %)[2156.62/(2156.62 + 18258.62]
= 0.5264% (exact value 0.5236%)
ia = (1 + 0.005264)12 1 = 0.0650 or 6.50% (exact value 6.467%)

7-41
$2,000 = $91.05 (P/A, i*, 30)
(P/A, i*, 30) = $2,000/$91.05 = 21.966
(P/A, 1%, 30) I
22.396
2
20.930
2
imo = 2% + (%) [(22.396 21.966)/(22.396 20.930)]
= 2.15% per month
Nominal ROR received by finance company = 12 (2.15%) = 25.8%

21

Homework Solutions for Engineering Economic Analysis, 10th Edition


Newnan, Lavelle, Eschenbach

7-42
$3,000 = $118.90 (P/A, i*, 36)
(P/A, i*, 36) = $3,000/$118.90 = 26.771
(P/A, i%, 36) i
27.661
1 %
26.543
1 %
imo = 1 % + % [(27.661 26.771)/(27.661 26.543)]
= 1.699% per month
Nominal Annual ROR = 12 (1.699%) = 20.4%

7-43
$15,000

A = $80
$9,000

PW of Benefits PW of Cost = $0
$15,000 (P/F, i%, 4) $9,000 $80 (P/A, i%, 4) = $0
Try i = 12%
$15,000 (0.6355) $9,000 $80 (3.037) = +$289.54
Try i = 15%
$15,000 (0.5718) $9,000 $80 (2.855) = $651.40
Performing Linear Interpolation:
i* = 12% + (3%) [289.54/(289.54 + 651.40)]
= 12.92%

22

Homework Solutions for Engineering Economic Analysis, 10th Edition


Newnan, Lavelle, Eschenbach

7-44
$65,000

$5,000

$240,000

$240,000 = $65,000 (P/A, i%, 13) $5,000 (P/G, i%, 13)


Try i = 15%
$65,000 (5.583) $5,000 (23.135) = $247,220 > $240,000
Try i = 18%
$65,000 (4.910) $5,000 (18.877) = $224,465 < $240,000
Rate of Return = 15% + 3% [($247,220 $240,000)/($247,220 $224,765)]
= 15.96%

7-45
(a) Total Annual Revenues = $500 (12 months) (4 apt.) = $24,000
Annual Revenues Expenses = $24,000 $8,000 = $16,000
To find Internal Rate of Return the Net Present Worth must be $0.
NPW = $16,000 (P/A, i*, 5) + $160,000 (P/F, i*, 5) $140,000
At i = 12%, NPW = $8,464
At i = 15%, NPW = $6,816
IRR

= 12% + (3%) [$8,464/($8,464 + $6,816)]


= 13.7%

(b) At 13.7% the apartment building is more attractive than the other options.

23

Homework Solutions for Engineering Economic Analysis, 10th Edition


Newnan, Lavelle, Eschenbach

7-46
NPW = $300,000 + $20,000 (P/F, i*, 10)
+ ($67,000 $3,000) (P/A, i*, 10) $600 (P/G, i*, 10)
Try i = 10%
NPW = $300,000 + $20,000 (0.3855) + ($64,000) (6.145)
$600 (22.891)
= $87,255 > $0
The interest rate is too low.
Try i = 18%
NPW = $300,000 + $20,000 (0.1911) + ($64,000) (4.494)
$600 (14.352)
= $17,173 < $0
The interest rate is too high.
Try i =15%
NPW = $300,000 + $20,000 (0.2472) + ($64,000) (5.019)
$600 (16.979)
= $9,130 > $0
Thus, the rate of return (IRR) is between 15% and 18%. By linear interpolation:
i* = 15% + (3%) [$9,130/($9,130 $17,173)]
= 16.0%

24

Homework Solutions for Engineering Economic Analysis, 10th Edition


Newnan, Lavelle, Eschenbach

7-47
g = 10%
A1 = $1,100
n = 20
i=?
P = $20,000

The payment schedule represents a geometric gradient.


There are two possibilities:
i g and i = g
Try the easier i = g computation first:
P = A1n (1 + i)1 where g = i = 0.10
$20,000 = $1,100 (20) (1.10)1 = $20,000
Rate of Return i* = g = 10%

7-48
(a) When n = , i = A/P = $3,180/$100,000 = 3.18%
(b) (A/P, i%, 100) = $3180/$100,000 = 0.318
From interest tables, i* = 3%
(c) (A/P, i%, 50) = $3, 180/$100,000 = 0.318
From interest tables, i* = 2%
The saving in water truck expense is just a small part of the benefits of the pipeline.
Convenience, improved quality of life, increased value of the dwellings, etc., all are
benefits. Thus, the pipeline appears justified.

25

Homework Solutions for Engineering Economic Analysis, 10th Edition


Newnan, Lavelle, Eschenbach

7-49
$800
$400

$6,000

$9,000

Year
0
14
58
9

Cash Flow
$9,000
+$800
+$400
+$6,000

PW of Cost = PW of Benefits
$9,000 = $400 (P/A, i%, 8) + $400 (P/A, i%, 4) + $6,000 (P/F, i%, 9)
Try i = 3%
$400 (7.020) + $400 (3.717) + $6,000 (0.7664) = $8,893 < $9,000
Try i = 2 %
$400 (7.170) + $400 (3.762) + $6,000 (0.8007) = $9,177 > $9,000
Rate of Return = 2 % + (1/2%) [($9,177 $9,000)/($9,177 $8,893)]
= 2.81%

26

Homework Solutions for Engineering Economic Analysis, 10th Edition


Newnan, Lavelle, Eschenbach

7-50
$12,000

$6,000
$3,000
n = 10

n = 10

n = 20

$28,000

PW of Cost = PW of Benefits
$28,000 = $3,000 (P/A, i%, 10) + $6,000 (P/A, i%, 10) (P/F, i%, 10) +
$12,000 (P/A, i%, 20) (P/F, i%, 20)
Try i = 12%
$3,000 (5.650) + $6,000 (5.650) (0.3220) + $12,000 (7.469) (0.1037)
= $37,160 > $28,000
Try i = 15%
$3,000 (5.019) + $6,000 (5.019) (0.2472) + $12,000 (6.259) (0.0611)
= $27,090 < $28,000
Performing Linear Interpolation:
i* = 15% (3%) [($28,000 $27,090)/($37,160 $27,090)]
= 15% (3%) (910/10,070)
= 14.73%

27

Homework Solutions for Engineering Economic Analysis, 10th Edition


Newnan, Lavelle, Eschenbach

7-51
This is a thought-provoking problem for which there is no single answer. Two
possible solutions are provided below.
(a) Assuming the MS degree is obtained by attending graduate school at night while
continuing with a full-time job:
MS
Degree

A = $3,000

n = 10
$1,500 $1,500

Cost: $1,500 per year for 2 years


Benefit: $3,000 per year for 10 years
Computation as of award of MS degree:
$1,500 (F/A, i%, 2) = $3,000 (P/A, i%, 10)
i* > 60
(b) Assuming the MS degree is obtained by one of year of full-time study
Cost: Difference between working & going to school. Whether working or at
school there are living expenses. The cost of the degree might be $24,000
Benefit: $3,000 per year for 10 years
$24,000 = $3,000 (P/A, i%, 10)
i* = 4.3%

28

Homework Solutions for Engineering Economic Analysis, 10th Edition


Newnan, Lavelle, Eschenbach

7-52
The problem requires an estimate for n- the expected life of the infant. Seventy or
seventy-five years might be the range of reasonable estimates. Here we will use 71
years.
The purchase of a $200 life subscription avoids the series of beginning-of-year
payments of $12.90. Based on 71 beginning-of-year payments,
A = $12.90
.
n = 70
$200

$200 - $12.90 = $12.90 (P/A, i%, 70)


(P/A, i%, 70) = $187.10/$ 12.90 = 14.50
6% < i* < 8%. By Calculator: i* = 6.83%

7-53
Year Case 1 (incl. Deposit)
0
$39,264.00
1
+$599.00
2
+$599.00
3
+$599.00
4
+$599.00
5
+$599.00
6
+$599.00
7
+$599.00
8
+$599.00
9
+$599.00
10
+$599.00
11
+$599.00
12
+$599.00

+$599.00
33
+$599.00
34
+$599.00
35
+$599.00
36
+$27,854.00 $625.00 = +$27,229.00
IRR = 0.86%
Nominal IRR = 10.32%
Effective IRR =10.83%
29

Homework Solutions for Engineering Economic Analysis, 10th Edition


Newnan, Lavelle, Eschenbach

7-54
The number of months between August 15 and January 15 is 5.
Month
0
1
2
3
4
5

Annual
Permit
$100
0
0
0
0
0

Semester
Permit
$65
0
0
0
0
65

To solve for the monthly interest rate set the two PWs equal to each other, so,
100 65
0.53846 .
100 = 65 65 (P/F, i, 5) . Thus, (1 i) 5
65
Solving get i = 0.1318 or 13.18% and ia = (1 + 0.1318)12 1 = 3.418 or 342%.
Unless the student is graduating in January or just doesnt have the $100, it is
clearly better to buy the permit a year at a time.

7-55
Details will vary by university, but is solved like Problem 7-54.

7-56
Annual
Quarter Payment
0
$65,000
1
0
2
0
3
0

Quarter
Payment
$18,000
18,000
18,000
18,000

To solve for the monthly interest rate set the two PWs equal to each other, so,
65000 = 18000 18000 (P/A, i, 3) . Thus, (P/A, i, 3) = 2.611 and interpolating
2.624 2.611
i = 7% + (1%)
= 7.28%, so, r = 4 x 0.0728 = 0.2912 or 29.1% and
2.624 2.577

ia = (1 + 0.0728)4 1 = 0.3246 or 32.5%. This is a high rate of return, but some firms
use an even higher hurdle rate for projects.

30

Homework Solutions for Engineering Economic Analysis, 10th Edition


Newnan, Lavelle, Eschenbach

7-57
$65,000 = $18,000( 1 + (P/A, i , 3))
The amount that the series of future payments is worth is:
65000 + 18000 = 47000 = 18000*(P/A, i , 3)
Using the end-of-period designation (default) in RATE (Excel) yields:
RATE(3,18000,-47000) = 7.2766%
One could also solve with quarterly payments at the beginning of the period:
RATE(4,18000,-65000,0,1) = 7.2766%

7-58
Insurance payments must be paid in advance, here on the first of the month or year.
Annual
Monthly
Month Basis
Basis
0
$1,650
$150
1
0
150
2
0
150
3
0
150
4
0
150
5
0
150
6
0
150
7
0
150
8
0
150
9
0
150
10
0
150
11
0
150
To solve for the monthly interest rate set the PWs of the two cash flows equal to
each other. Thus, 1650 = 150 150 (P/A, i, 11), so, (P/A, i, 11) = 10.0.
Interpolating
10.071 10.0
= 1.624%. Next, ia = (1 + 0.01624)12 1 = 0.2133
i = 1.5% + (0.25%)

10.071 9.928
or 21.3%. This is a relatively high rate of return, but the student might prefer to pay
monthly if there is a significant chance of wrecking the car before the year is up.

7-59
Details will vary by student, but solved like Problem 7-58.

31

Homework Solutions for Engineering Economic Analysis, 10th Edition


Newnan, Lavelle, Eschenbach

7-60
Year
0
1 3
Computed ROR

A
$2,000
+$800
9.7%

B
$2,800
+$1,100
8.7%

(B- A)
$800
+$300
6.1%

The rate of return on the increment (B- A) exceeds the Minimum Attractive Rate of
Return (MARR), therefore the higher cost alternative B should be selected.

7-61
Year
0
1
2
3
4
Computed ROR

X
$100
+$35
+$35
+$35
+$35
15.0%

Y
$50
+$16.5
+$16.5
+$16.5
+$16.5
12.1%

X- Y
$50
+$18.5
+$18.5
+$18.5
+$18.5
17.8%

The ROR on X- Y is greater than 10%. Therefore, the increment is desirable.


Select X.

7-62
Year
0
1- 10
Computed ROR

A
$100.00
+$19.93
15%

B
$50.00
+$11.93
20%

ROR = 9.61% > MARR.


Select A.

32

(B- A)
$50.00
+$8.00
9.61%

Homework Solutions for Engineering Economic Analysis, 10th Edition


Newnan, Lavelle, Eschenbach

7-63
Year
0
1
2
3
4
Computed ROR

X
$5,000
$3,000
+$4,000
+$4,000
+$4,000
16.9%

Y
$5,000
+$2,000
+$2,000
+$2,000
+$2,000
21.9%

X- Y
$0
$5,000
+$2,000
+$2,000
+$2,000
9.7%

Since X- Y difference between alternatives is desirable, select Alternative X.

7-64
(a) Present Worth Analysis- Maximize NPW
NPWA = $746 (P/A, 8%, 5) $2,500
= $746 (3.993) $2,500 = +$479
NPWB = $1,664 (P/A, 8%, 5) $6,000 = +$644
Select B.
(b) Annual Cash Flow Analysis- Maximize (EUAB- EUAC)
(EUAB- EAUC)A = $746 $2,500 (A/P, 8%, 5)
= $746 $2,500 (0.2505)
= +$120
(EUAB EUAC)B = $1,664 $6,000 (A/P, 8%, 5)
= +$161
Select B.
(c) Rate of Return Analysis: Compute the rate of return on the B- A increment of
investment and compare to 8% MARR.
Year A
B
B- A
0
$2,500 $6,000 $3,500
15 +$746
+$1,664 +$918
$3,500 =$918 (P/A, i%, 5)
Try i = 8%, $918 (3.993) = $3,666 > $3,500
Try i = 10%, $918 (3.791) = $3,480 < $3,500
Rate of Return = 9.8%
Since ROR > MARR, B- A increment is desirable. Select B.

33

Homework Solutions for Engineering Economic Analysis, 10th Edition


Newnan, Lavelle, Eschenbach

7-65
Year
Alt. A
Alt. B
A-B

0
-12000
-3000
-9000

-3000
3000

-3000
3000

-3000
3000

-3000
3000

-3000
3000

-3000
3000

-3000
3000

8
1200
1200

IRR of A B stream = IRR (the A B values for the Years 08) = 27.90%
Since ROR > MARR (15%),
choose the higher initial cost alternative, A (purchasing the equipment).

7-66
First Cost
Maintenance &
Operating Costs
Annual Benefit
Salvage Value

B
$300,000
$25,000

A
$615,000
$10,000

A- B
$315,000
$15,000

$92,000
$5,000

$158,000
$65,000

$66,000
$70,000

NPW = $315,000 + [$66,000 ($15,000)] (P/A, i*, 10) + $70,000 (P/F, i*, 10) = $0
Try i = 15%
$315,000 + [$66,000 ($15,000)] (5.019) + $70,000 (0.2472) = $108,840
ROR > MARR (15%)
The higher cost alternative A is the more desirable alternative.

34

Homework Solutions for Engineering Economic Analysis, 10th Edition


Newnan, Lavelle, Eschenbach

7-67
Year

(A) Gas Station

0
120
Computed ROR

$80,000
+$8,000
7.75%

(B) Ice Cream


Stand
$120,000
+$11,000
6.63%

(B- A)
$40,000
+$3,000
4.22%

The rate of return in the incremental investment (B- A) is less than the desired 6%.
In this situation the lower cost alternative (A) Gas Station should be selected.

7-68
MARR = 5%

P = $30,000 n = 35 years

Alternative 1: Withdraw $15,000 today and lose $15,000


Alternative 2: Wait, leave your fund in the system until retirement.
Equivalency seeks to determine what future amount is equal to $15,000 now.
F = P (1 + i)n
= $30,000 (1.05)35
= $30,000 (5.516015)
= $165,480.46
Therefore:
$15,000 = $165,480.46 (1 + i)35
$15,000 (1 + i)35 = $165,480.46
(1 + i) = [(165,480.46/$15,000)]1/35
i = 1.071 1 = 7.1002% > 5%
Unless $15,000 can be invested with a return higher than 7.1%, it is better to wait for
35 years for the retirement fund. $15,000 now is only equivalent to $165,480.46 35
years from now if the interest rate now is 7.1% instead of the quoted 5%.

35

Homework Solutions for Engineering Economic Analysis, 10th Edition


Newnan, Lavelle, Eschenbach

7-69
(a)
$150

A = $100
.
n = 20

$2,000

($2,000 $150) = $100 (P/A, i%, 20)


(P/A, i%, 20) = $1,850/$100 = 18.5
I = % per month
The alternatives are equivalent at a nominal 9% annual interest.
(b) Take Alt 1- the $2,000- and invest the money at a higher interest rate.

7-70
(a) Salvage = 0.15 x $380,000 = $57,000 and firms interest rate = 12%.
Year

Purchase

Lease

0
1
2
3
4
5
6

$380,000
0
0
0
0
0
57,000

$60,000
60,000
60,000
60,000
60,000
60,000
0

Purchase
Lease
$320,000
60,000
60,000
60,000
60,000
60,000
57,000

NPW = 0 = 320000 + 60000 (P/A, IRR , 5) + 57000 (P/A, IRR, 6) and


interpolating
2538

= 3.24% (also 3.24% from Excel). The IRR is


IRR = 3% + (0.5%)
2538 2730
well below the firms interest rate on the borrowed amount ($320,000) from
leasing, so lease the bulldozer.

36

Homework Solutions for Engineering Economic Analysis, 10th Edition


Newnan, Lavelle, Eschenbach

(b) The firm receives $65,000 more than it spends on operating and maintenance
costs.
Year

Purchase

Lease

0
1
2
3
4
5
6

$380,000
65,000
65,000
65,000
65,000
65,000
65,000
57,000

$60,000
60,000
60,000
60,000
60,000
60,000
0

Purchase
Lease
$320,000
125,000
125,000
125,000
125,000
125,000
122,000

NPW = 0 = -320000 + 125000 (P/A, IRR, 5) + 122000 (P/F, IRR, 6) and


interpolating
9778

IRR = 30% + (5%)


= 31.5% (31.42% from Excel). Clearly, the
9778 22346
situation has changed. The interest rate on the borrowed amount is now well
above the firms interest rate, so, buy the bulldozer. The rate of return for the
bulldozer will clearly be largest for this cash flow and is given by
PW = 0 = 380000 + 65000 (P/A, ROR, 6) + 57000 (P/F, ROR, 6) and
interpolating
5777
= 4.43% (4.43% from Excel).
ROR = 4% + (0.5%)
5777 960
Note that the author has failed to give a practical scenario for how the $65,000
benefit can be realized if the bulldozer is purchased instead of leased!

37

Homework Solutions for Engineering Economic Analysis, 10th Edition


Newnan, Lavelle, Eschenbach

7-71
(a) Salvage = $50,000 and communitys interest rate = 8%.
Year

Purchase

Lease

0
1
2
3
4
5
6
7
8
9
10

$480,000
0
0
0
0
0
0
0
0
0
50,000

$70,000
70,000
70,000
70,000
70,000
70,000
70,000
70,000
70,000
70,000
0

Purchase
Lease
$410,000
70,000
70,000
70,000
70,000
70,000
70,000
70,000
70,000
70,000
50,000

NPW = 0 = 410000 +70000 (P/A, IRR , 9) + 50000 (P/A, IRR, 10) and
interpolating
12405

= 10.74% (10.71% Excel). The IRR is above


IRR = 10% + (2%)
12405 20940
the communitys interest rate on the borrowed amount ($410,000) from leasing,
so buy the generator.
(b) The community spends $80,000 less on fuel and maintenance than it spends on
buying power.
Year

Purchase

Lease

0
1
2
3
4
5
6
7
8
9
10

$480,000
80,000
80,000
80,000
80,000
80,000
80,000
80,000
80,000
80,000
80,000
50,000

$70,000
70,000
70,000
70,000
70,000
70,000
70,000
70,000
70,000
70,000
0

Purchase
Lease
$410,000
150,000
150,000
150,000
150,000
150,000
150,000
150,000
150,000
150,000
130,000

38

Homework Solutions for Engineering Economic Analysis, 10th Edition


Newnan, Lavelle, Eschenbach

NPW = 0 = 410000 + 150000 (P/A, IRR, 9) + 130000 (P/F, IRR, 10) and
interpolating
52275

IRR = 30% + (5%)


= 34.66% (34.63% from Excel). The interest
52275 3789
rate on the borrowed amount is now well above the firms interest rate, so buy
the generator. The rate of return for the generator will clearly be largest for this
cash flow and is given by
PW = 0 = 480000 + 80000 (P/A, ROR, 10) + 50000 (P/F, ROR, 10) and
interpolating
30875

ROR = 10% + (2%)


= 11.44% (11.42% from Excel).
30875 11900
Note that the author has failed to give a practical scenario for how the $80,000
benefit can be realized if the generator is purchased instead of leased!

7-72
Year
0
1
2
3
4

A
$9,200
+$1,850
+$1,850
+$1,850
+$1,850

5
6
7
8

+$1,850
+$1,850
+$1,850
+$1,850

B
$5,000
+$1,750
+$1,750
+$1,750
+$1,750
$5,000
+$1,750
+$1,750
+$1,750
+$1,750

A- B
$4,200
+$100
+$100
+$100
+$5,100

NPW at 7%
$4,200
+$93
+$87
+$82
+$3,891

NPW at 9%
$4,200
+$92
+$84
+$77
+$3,613

+$100
+$100
+$100
+$100
Sum

+$71
+$67
+$62
+$58
+$211

+$65
+$60
+$55
+$50
$104

ROR 8.3%
Choose Alternative A.

39

Homework Solutions for Engineering Economic Analysis, 10th Edition


Newnan, Lavelle, Eschenbach

7-73
Year
0
1
2

Zappo
$56
$56
$0

Kicko
$90
$0
$0

Kicko Zappo
$34
+$56
$0

Compute the incremental rate of return on (Kicko Zappo)


PW of Cost = PW of Benefit
$34 = $56 (P/F, i%, 1)
(P/F, i%, 1) = $34/$56 = 0.6071
From interest tables, incremental rate of return > 60% (ROR = 64.7%), hence the
increment of investment is desirable.
Buy Kicko.

7-74
Year
0
1
2
3
4
5
6
7
8

A
$9,200
+$1,850
+$1,850
+$1,850
+$1,850
+$1,850
+$1,850
+$1,850
+$1,850

B
$5,000
+$1,750
+$1,750
+$1,750
+$1,750 $5,000
+$1,750
+$1,750
+$1,750
+$1,750

A- B
$4,200
+$100
+$100
+$100
+$100 +$5,000
+$100
+$100
+$100
+$100
Sum

Rates of Return
A: $9,200 = $1,850 (P/A, i%, 5)
Rate of Return = 11.7%
B: $5,000 = $1,750 (P/A, i%, 4)
Rate of Return = 15%
AB: $4,200 = $100 (P/A, i%, 8) + $5,000 (P/F, i%, 4)
RORA-B = 8.3%
Select A.

40

Homework Solutions for Engineering Economic Analysis, 10th Edition


Newnan, Lavelle, Eschenbach

7-75
Year
0
1- 10
11- 15
15
Computed ROR

A
$150
+$25
+$25
+$20
14.8%

B
$100
+$22.25
$0
$0
18%

A- B
$50
+$2.75
+$25
+$20
11.6%

Rate of Return (A- B):


$50 = $2.75 (P/A, i%, 10) + $25 (P/A, i%, 5) (P/F, i%, 10) + $20 (P/F, i%, 15)
Rate of Return = 11.65
Select A.

7-76
This is an unusual problem with an extremely high rate of return. Available interest
tables obviously are useless.
One may write:
PW of Cost = PW of Benefits
$0.5 = $3.5 (1 + i)1 + $0.9 (1 + i)2 + $3.9 (1 + i)3 + $8.6 (1 + i)4 +
For high interest rates only the first few terms of the series are significant:
Try i = 650%
PW of Benefits = $3.5/(1 + 6.5) + $0.9/(1 + 6.5)2 + $3.9/(1 + 6.5)3 + $8.6/(1 + 6.5)4 +

= 0.467 + 0.016 + 0.009 + 0.003


= 0.495
Try i = 640%
PW of Benefits = $3.5/(1 + 6.4) + $0.9/(1 + 6.4)2 + $3.9/(1 + 6.4)3 + $8.6/(1 + 6.4)4 +

= 0.473 + 0.016 + 0.010 + 0.003


= 0.502
i* = 642%
(Calculator Solution: i = 642.9%)

41

Homework Solutions for Engineering Economic Analysis, 10th Edition


Newnan, Lavelle, Eschenbach

7-77
$52,000.00
3.00%
10.00%
15
4.00%

income
income gradient
% deposit
horizon (years)
savings rate

Year
1
2
3
4
5
6
7
8
9
10
11
12
13
14
15

Cumulative
Salary
Deposit
Savings
$52,000.00 $5,200.00 $5,200.00
53,560.00 5,356.00 10,764.00
55,166.80 5,516.68 16,711.24
56,821.80 5,682.18 23,061.87
58,526.46 5,852.65 29,836.99
60,282.25 6,028.23 37,058.70
62,090.72 6,209.07 44,750.12
63,953.44 6,395.34 52,935.46
65,872.04 6,587.20 61,640.09
67,848.21 6,784.82 70,890.51
69,883.65 6,988.37 80,714.50
71,980.16 7,198.02 91,141.09
74,139.57 7,413.96 102,200.69
76,363.75 7,636.38 113,925.10
78,654.67 7,865.47 126,347.57

For any row: Salary = (1 + 0.03)*(Previous years Salary)


Deposit = (Percent Deposit)*(Current years Salary)
Savings = (1 + 0.04)*(Previous years Savings) + Current years Deposit
Amount saved is $126,347.57 in 15 years.

7-78
$55,000.00
2.00%
10.00%
40
5.00%

income
income gradient
% deposit
horizon (years)
savings rate

42

Homework Solutions for Engineering Economic Analysis, 10th Edition


Newnan, Lavelle, Eschenbach

Year
1
2
3
4
5
6
7
8
9
10
11
12
13
14
15
16
17
18
19
20
21
22
23
24
25
26
27
28
29
30
31
32
33
34
35
36
37
38
39
40

Salary
$55,000.00
56,100.00
57,222.00
58,366.44
59,533.77
60,724.44
61,938.93
63,177.71
64,441.27
65,730.09
67,044.69
68,385.59
69,753.30
71,148.36
72,571.33
74,022.76
75,503.21
77,013.28
78,553.54
80,124.61
81,727.11
83,361.65
85,028.88
86,729.46
88,464.05
90,233.33
92,038.00
93,878.76
95,756.33
97,671.46
99,624.89
101,617.38
103,649.73
105,722.73
107,837.18
109,993.93
112,193.80
114,437.68
116,726.43
119,060.96

Deposit
$5,500.00
5,610.00
5,722.20
5,836.64
5,953.38
6,072.44
6,193.89
6,317.77
6,444.13
6,573.01
6,704.47
6,838.56
6,975.33
7,114.84
7,257.13
7,402.28
7,550.32
7,701.33
7,855.35
8,012.46
8,172.71
8,336.16
8,502.89
8,672.95
8,846.40
9,023.33
9,203.80
9,387.88
9,575.63
9,767.15
9,962.49
10,161.74
10,364.97
10,572.27
10,783.72
10,999.39
11,219.38
11,443.77
11,672.64
11,906.10

Cumulative
Savings
$5,500.00
11,385.00
17,676.45
24,396.92
31,570.14
39,221.09
47,376.04
56,062.61
65,309.87
75,148.37
85,610.26
96,729.33
108,541.13
121,083.02
134,394.30
148,516.30
163,492.43
179,368.38
196,192.15
214,014.22
232,887.65
252,868.19
274,014.49
296,388.16
320,053.97
345,080.01
371,537.81
399,502.57
429,053.33
460,273.15
493,249.29
528,073.49
564,842.14
603,656.52
644,623.07
687,853.61
733,465.67
781,582.72
832,334.50
885,857.33

For any row: Salary = (1 + 0.02)*(Previous years Salary)


Deposit = (Percent Deposit)*(Current years Salary)
43

Homework Solutions for Engineering Economic Analysis, 10th Edition


Newnan, Lavelle, Eschenbach

Savings = (1 + 0.05)*(Previous years Savings) + Current years Deposit


Amount saved is $885,857.33 in 40 years.

7-79
$55,000.00
2.00%
11.29%
40
5.00%

income
income gradient
% deposit
horizon (years)
savings rate

Year
1
2
3
4
5
6
7
8
9
10
11
12
13
14
15
16
17
18
19
20
21
22
23
24
25
26
27
28
29
30
31

Salary
$55,000.00
56,100.00
57,222.00
58,366.44
59,533.77
60,724.44
61,938.93
63,177.71
64,441.27
65,730.09
67,044.69
68,385.59
69,753.30
71,148.36
72,571.33
74,022.76
75,503.21
77,013.28
78,553.54
80,124.61
81,727.11
83,361.65
85,028.88
86,729.46
88,464.05
90,233.33
92,038.00
93,878.76
95,756.33
97,671.46
99,624.89

Deposit
$6,209.50
6,333.69
6,460.36
6,589.57
6,721.36
6,855.79
6,992.91
7,132.76
7,275.42
7,420.93
7,569.35
7,720.73
7,875.15
8,032.65
8,193.30
8,357.17
8,524.31
8,694.80
8,868.70
9,046.07
9,226.99
9,411.53
9,599.76
9,791.76
9,987.59
10,187.34
10,391.09
10,598.91
10,810.89
11,027.11
11,247.65

Cumulative
Savings
$6,209.50
12,853.67
19,956.71
27,544.12
35,642.69
44,280.61
53,487.55
63,294.69
73,734.84
84,842.51
96,653.98
109,207.41
122,542.93
136,702.73
151,731.17
167,674.90
184,582.96
202,506.90
221,500.94
241,622.06
262,930.15
285,488.19
309,362.36
334,622.23
361,340.94
389,595.33
419,466.18
451,038.40
484,401.21
519,648.38
556,878.45
44

Homework Solutions for Engineering Economic Analysis, 10th Edition


Newnan, Lavelle, Eschenbach

32
33
34
35
36
37
38
39
40

101,617.38
103,649.73
105,722.73
107,837.18
109,993.93
112,193.80
114,437.68
116,726.43
119,060.96

11,472.60
11,702.05
11,936.10
12,174.82
12,418.31
12,666.68
12,920.01
13,178.41
13,441.98

596,194.97
637,706.78
681,528.21
727,779.44
776,586.73
828,082.74
882,406.90
939,705.66
1,000,132.92

For any row: Salary = (1 + 0.02)*(Previous years salary)


Deposit = (Percent deposit)*(Current years salary)
Savings = (1 + 0.05)*(Previous years savings) + Current years deposit
To solve, just vary the percent deposit to get $1M in savings for year 40.
Amount saved is $1,000,132.92 in 40 years at 11.29%.

7-80
Details will vary by student, but solved like Problem 7-79.

45

Homework Solutions for Engineering Economic Analysis, 10th Edition


Newnan, Lavelle, Eschenbach

46

Das könnte Ihnen auch gefallen